LSAT and Law School Admissions Forum

Get expert LSAT preparation and law school admissions advice from PowerScore Test Preparation.

User avatar
 Dave Killoran
PowerScore Staff
  • PowerScore Staff
  • Posts: 5848
  • Joined: Mar 25, 2011
|
#43482
Complete Question Explanation
(The complete setup for this game can be found here: lsat/viewtopic.php?t=7431)

The correct answer choice is (A)

The condition in the rule forces K to be assigned to T, M to be assigned to R, and J to be assigned to V. The only choice remaining is whether F or G teams with S:
J95_Game_#4_#24_diagram 1.png
Answer choice (A) is therefore correct.

Get the most out of your LSAT Prep Plus subscription.

Analyze and track your performance with our Testing and Analytics Package.